subject
Mathematics, 10.03.2021 05:40 akkira02

A. 109°
B. 34.5°
C. 62.5°
D. 69°

Show work, if possible. Thanks❤️


Help I will be marking brainliest!!!

A. 109°
B. 34.5°
C. 62.5°
D. 69°
Show work, if possible. Tha

ansver
Answers: 3

Another question on Mathematics

question
Mathematics, 20.06.2019 18:04
For an experiment, a penny is tossed. at the same time, a spinner with 8 congruent sections numbered 1–8 is spun. how many possible outcomes are there for the experiment?
Answers: 1
question
Mathematics, 21.06.2019 14:30
Which quadratic function best fits this data? x y 1 32 2 78 3 178 4 326 5 390 6 337 a. y=11.41x2+154.42x−143.9 b. y=−11.41x2+154.42x−143.9 c. y=11.41x2+154.42x+143.9 d. y=−11.41x2+154.42x+143.9
Answers: 3
question
Mathematics, 21.06.2019 14:30
Which sentence describes why polygon mnop is congruent to polygon jklp? a. polygon jklp maps to polygon mnop through a translation. b. polygon jklp maps to polygon mnop through a rotation. c. polygon jklp maps to polygon mnop through a dilation. d. polygon jklp maps to polygon mnop through a stretch.
Answers: 2
question
Mathematics, 21.06.2019 19:10
How many solutions does the nonlinear system of equations graphed below have?
Answers: 2
You know the right answer?
A. 109°
B. 34.5°
C. 62.5°
D. 69°

Show work, if possible. Thanks❤️
Questions
question
Social Studies, 26.03.2020 21:02
question
Mathematics, 26.03.2020 21:02
question
Mathematics, 26.03.2020 21:02
question
Mathematics, 26.03.2020 21:02
question
Mathematics, 26.03.2020 21:02
Questions on the website: 13722362